Skip to Main Content

PrepTest 79, Game 2, Question 8

Transcript

If exactly one ranger is assigned to area 1, then which one of the following must be true? So we're gonna only have one ranger in area 1. Now if we only have 1 ranger in area 1, we know for sure that, that one ranger is not going to be L. Under no circumstances can L occur or be assigned to an area alone, since L must go with M or K.

So we can eliminate L here from area 1. Now this question lets us combine lots of other rules, or some of our other rules as well. So if L cannot go in 1 and P and O can not go in 1 and of course M is not going in 1. We've eliminated L, M, O and P from this area, area 1. That leaves just J or K left that can go in area 1.

Now, only one of J or K is going in area 1, which means that J and K are not going to be assigned to the same area. So if J and K are not assigned to the same area, that means that O is not in 2. So whenever we put O in area 2, then J and K will become sort of a pair, and be assigned to the same committee. Since there's only one available spot to be assigned in area 1 and one of J or K must go there, they are not going to occur together, which means that O cannot be in 2.

This is particularly helpful since O also can't go in 1. So we know that O is gonna end up down here with M. So, this is a must be true question, so the correct answer is D. O is assigned to area 3. So, we already had O excluded from area 1 and then we figure out that O could not go in area 2, since one, or J or K needs to go on 1.

So there's only one other placement for O and that is in area 3. So this is our correct answer.

Read full transcript